Moment of inertia of a solid right circular cone












1














So I was trying to do this using triple integrals (I know there are simpler ways) when I encountered something I can't really justify..here it is



the mass is uniformly distributed so $dm=rho dv$ and the equation of the cone is $$ x^2 + y^2 = frac{R^2}{h^2}z^2$$
where R is the radius of circular top of the cone and h is it's height , now the square of the distance from the z axis is $x^2 + y^2 $ and I evaluate the integral using cylindrical coordinates and the equation becomes $r=frac{R}{h}z$ (taking the positive root)



$$iiint_v r^3rho,dz,dr,dtheta$$ where the limits of z are $z=frac{h}{R}r$ and h , r from zero to R and $theta$ from zero to $2pi$



I do the integral and get the correct answer which $frac{3MR^2}{10}$



here's my question..when I replace the $r^3$ term in the integrand with $r^3=frac{R^3}{h^3}z^3$ and do the integral (using the same limits and order of integration) I get an incorrect result and I can't explain why this happens










share|cite|improve this question



























    1














    So I was trying to do this using triple integrals (I know there are simpler ways) when I encountered something I can't really justify..here it is



    the mass is uniformly distributed so $dm=rho dv$ and the equation of the cone is $$ x^2 + y^2 = frac{R^2}{h^2}z^2$$
    where R is the radius of circular top of the cone and h is it's height , now the square of the distance from the z axis is $x^2 + y^2 $ and I evaluate the integral using cylindrical coordinates and the equation becomes $r=frac{R}{h}z$ (taking the positive root)



    $$iiint_v r^3rho,dz,dr,dtheta$$ where the limits of z are $z=frac{h}{R}r$ and h , r from zero to R and $theta$ from zero to $2pi$



    I do the integral and get the correct answer which $frac{3MR^2}{10}$



    here's my question..when I replace the $r^3$ term in the integrand with $r^3=frac{R^3}{h^3}z^3$ and do the integral (using the same limits and order of integration) I get an incorrect result and I can't explain why this happens










    share|cite|improve this question

























      1












      1








      1







      So I was trying to do this using triple integrals (I know there are simpler ways) when I encountered something I can't really justify..here it is



      the mass is uniformly distributed so $dm=rho dv$ and the equation of the cone is $$ x^2 + y^2 = frac{R^2}{h^2}z^2$$
      where R is the radius of circular top of the cone and h is it's height , now the square of the distance from the z axis is $x^2 + y^2 $ and I evaluate the integral using cylindrical coordinates and the equation becomes $r=frac{R}{h}z$ (taking the positive root)



      $$iiint_v r^3rho,dz,dr,dtheta$$ where the limits of z are $z=frac{h}{R}r$ and h , r from zero to R and $theta$ from zero to $2pi$



      I do the integral and get the correct answer which $frac{3MR^2}{10}$



      here's my question..when I replace the $r^3$ term in the integrand with $r^3=frac{R^3}{h^3}z^3$ and do the integral (using the same limits and order of integration) I get an incorrect result and I can't explain why this happens










      share|cite|improve this question













      So I was trying to do this using triple integrals (I know there are simpler ways) when I encountered something I can't really justify..here it is



      the mass is uniformly distributed so $dm=rho dv$ and the equation of the cone is $$ x^2 + y^2 = frac{R^2}{h^2}z^2$$
      where R is the radius of circular top of the cone and h is it's height , now the square of the distance from the z axis is $x^2 + y^2 $ and I evaluate the integral using cylindrical coordinates and the equation becomes $r=frac{R}{h}z$ (taking the positive root)



      $$iiint_v r^3rho,dz,dr,dtheta$$ where the limits of z are $z=frac{h}{R}r$ and h , r from zero to R and $theta$ from zero to $2pi$



      I do the integral and get the correct answer which $frac{3MR^2}{10}$



      here's my question..when I replace the $r^3$ term in the integrand with $r^3=frac{R^3}{h^3}z^3$ and do the integral (using the same limits and order of integration) I get an incorrect result and I can't explain why this happens







      multivariable-calculus






      share|cite|improve this question













      share|cite|improve this question











      share|cite|improve this question




      share|cite|improve this question










      asked Nov 24 at 21:57









      Km356

      887




      887






















          1 Answer
          1






          active

          oldest

          votes


















          1














          Yes the correct set up is as follows



          $$int_0^{2pi},dthetaint_0^h dz int_0^{frac{R}{h}z} rho r^3,dr$$



          we can't substitute $r^3=frac{R^3}{h^3}z^3$ because the $r$ indicated here represent the maximum radius to be considered for any fixed $z$, that is $$r_{MAX}^3=frac{R^3}{h^3}z^3$$ but for any $z$ we have $0 le r le r_{MAX}$.






          share|cite|improve this answer























          • isn't the lower limit for z a function of r? the integral you wrote doesn't lead to the correct answer
            – Km356
            Nov 24 at 22:19










          • @Km356 Yes of course sorry, of course we need $r(z)$! I fix that
            – gimusi
            Nov 24 at 22:22










          • @Km356 That value is indeed the $r_{MAX}$ and that the reason why we can't substitute that with $r$ inside the integral.
            – gimusi
            Nov 24 at 22:23










          • yes that is indeed what I missed,, thanks
            – Km356
            Nov 24 at 22:25











          Your Answer





          StackExchange.ifUsing("editor", function () {
          return StackExchange.using("mathjaxEditing", function () {
          StackExchange.MarkdownEditor.creationCallbacks.add(function (editor, postfix) {
          StackExchange.mathjaxEditing.prepareWmdForMathJax(editor, postfix, [["$", "$"], ["\\(","\\)"]]);
          });
          });
          }, "mathjax-editing");

          StackExchange.ready(function() {
          var channelOptions = {
          tags: "".split(" "),
          id: "69"
          };
          initTagRenderer("".split(" "), "".split(" "), channelOptions);

          StackExchange.using("externalEditor", function() {
          // Have to fire editor after snippets, if snippets enabled
          if (StackExchange.settings.snippets.snippetsEnabled) {
          StackExchange.using("snippets", function() {
          createEditor();
          });
          }
          else {
          createEditor();
          }
          });

          function createEditor() {
          StackExchange.prepareEditor({
          heartbeatType: 'answer',
          autoActivateHeartbeat: false,
          convertImagesToLinks: true,
          noModals: true,
          showLowRepImageUploadWarning: true,
          reputationToPostImages: 10,
          bindNavPrevention: true,
          postfix: "",
          imageUploader: {
          brandingHtml: "Powered by u003ca class="icon-imgur-white" href="https://imgur.com/"u003eu003c/au003e",
          contentPolicyHtml: "User contributions licensed under u003ca href="https://creativecommons.org/licenses/by-sa/3.0/"u003ecc by-sa 3.0 with attribution requiredu003c/au003e u003ca href="https://stackoverflow.com/legal/content-policy"u003e(content policy)u003c/au003e",
          allowUrls: true
          },
          noCode: true, onDemand: true,
          discardSelector: ".discard-answer"
          ,immediatelyShowMarkdownHelp:true
          });


          }
          });














          draft saved

          draft discarded


















          StackExchange.ready(
          function () {
          StackExchange.openid.initPostLogin('.new-post-login', 'https%3a%2f%2fmath.stackexchange.com%2fquestions%2f3012140%2fmoment-of-inertia-of-a-solid-right-circular-cone%23new-answer', 'question_page');
          }
          );

          Post as a guest















          Required, but never shown

























          1 Answer
          1






          active

          oldest

          votes








          1 Answer
          1






          active

          oldest

          votes









          active

          oldest

          votes






          active

          oldest

          votes









          1














          Yes the correct set up is as follows



          $$int_0^{2pi},dthetaint_0^h dz int_0^{frac{R}{h}z} rho r^3,dr$$



          we can't substitute $r^3=frac{R^3}{h^3}z^3$ because the $r$ indicated here represent the maximum radius to be considered for any fixed $z$, that is $$r_{MAX}^3=frac{R^3}{h^3}z^3$$ but for any $z$ we have $0 le r le r_{MAX}$.






          share|cite|improve this answer























          • isn't the lower limit for z a function of r? the integral you wrote doesn't lead to the correct answer
            – Km356
            Nov 24 at 22:19










          • @Km356 Yes of course sorry, of course we need $r(z)$! I fix that
            – gimusi
            Nov 24 at 22:22










          • @Km356 That value is indeed the $r_{MAX}$ and that the reason why we can't substitute that with $r$ inside the integral.
            – gimusi
            Nov 24 at 22:23










          • yes that is indeed what I missed,, thanks
            – Km356
            Nov 24 at 22:25
















          1














          Yes the correct set up is as follows



          $$int_0^{2pi},dthetaint_0^h dz int_0^{frac{R}{h}z} rho r^3,dr$$



          we can't substitute $r^3=frac{R^3}{h^3}z^3$ because the $r$ indicated here represent the maximum radius to be considered for any fixed $z$, that is $$r_{MAX}^3=frac{R^3}{h^3}z^3$$ but for any $z$ we have $0 le r le r_{MAX}$.






          share|cite|improve this answer























          • isn't the lower limit for z a function of r? the integral you wrote doesn't lead to the correct answer
            – Km356
            Nov 24 at 22:19










          • @Km356 Yes of course sorry, of course we need $r(z)$! I fix that
            – gimusi
            Nov 24 at 22:22










          • @Km356 That value is indeed the $r_{MAX}$ and that the reason why we can't substitute that with $r$ inside the integral.
            – gimusi
            Nov 24 at 22:23










          • yes that is indeed what I missed,, thanks
            – Km356
            Nov 24 at 22:25














          1












          1








          1






          Yes the correct set up is as follows



          $$int_0^{2pi},dthetaint_0^h dz int_0^{frac{R}{h}z} rho r^3,dr$$



          we can't substitute $r^3=frac{R^3}{h^3}z^3$ because the $r$ indicated here represent the maximum radius to be considered for any fixed $z$, that is $$r_{MAX}^3=frac{R^3}{h^3}z^3$$ but for any $z$ we have $0 le r le r_{MAX}$.






          share|cite|improve this answer














          Yes the correct set up is as follows



          $$int_0^{2pi},dthetaint_0^h dz int_0^{frac{R}{h}z} rho r^3,dr$$



          we can't substitute $r^3=frac{R^3}{h^3}z^3$ because the $r$ indicated here represent the maximum radius to be considered for any fixed $z$, that is $$r_{MAX}^3=frac{R^3}{h^3}z^3$$ but for any $z$ we have $0 le r le r_{MAX}$.







          share|cite|improve this answer














          share|cite|improve this answer



          share|cite|improve this answer








          edited Nov 24 at 22:22

























          answered Nov 24 at 22:03









          gimusi

          1




          1












          • isn't the lower limit for z a function of r? the integral you wrote doesn't lead to the correct answer
            – Km356
            Nov 24 at 22:19










          • @Km356 Yes of course sorry, of course we need $r(z)$! I fix that
            – gimusi
            Nov 24 at 22:22










          • @Km356 That value is indeed the $r_{MAX}$ and that the reason why we can't substitute that with $r$ inside the integral.
            – gimusi
            Nov 24 at 22:23










          • yes that is indeed what I missed,, thanks
            – Km356
            Nov 24 at 22:25


















          • isn't the lower limit for z a function of r? the integral you wrote doesn't lead to the correct answer
            – Km356
            Nov 24 at 22:19










          • @Km356 Yes of course sorry, of course we need $r(z)$! I fix that
            – gimusi
            Nov 24 at 22:22










          • @Km356 That value is indeed the $r_{MAX}$ and that the reason why we can't substitute that with $r$ inside the integral.
            – gimusi
            Nov 24 at 22:23










          • yes that is indeed what I missed,, thanks
            – Km356
            Nov 24 at 22:25
















          isn't the lower limit for z a function of r? the integral you wrote doesn't lead to the correct answer
          – Km356
          Nov 24 at 22:19




          isn't the lower limit for z a function of r? the integral you wrote doesn't lead to the correct answer
          – Km356
          Nov 24 at 22:19












          @Km356 Yes of course sorry, of course we need $r(z)$! I fix that
          – gimusi
          Nov 24 at 22:22




          @Km356 Yes of course sorry, of course we need $r(z)$! I fix that
          – gimusi
          Nov 24 at 22:22












          @Km356 That value is indeed the $r_{MAX}$ and that the reason why we can't substitute that with $r$ inside the integral.
          – gimusi
          Nov 24 at 22:23




          @Km356 That value is indeed the $r_{MAX}$ and that the reason why we can't substitute that with $r$ inside the integral.
          – gimusi
          Nov 24 at 22:23












          yes that is indeed what I missed,, thanks
          – Km356
          Nov 24 at 22:25




          yes that is indeed what I missed,, thanks
          – Km356
          Nov 24 at 22:25


















          draft saved

          draft discarded




















































          Thanks for contributing an answer to Mathematics Stack Exchange!


          • Please be sure to answer the question. Provide details and share your research!

          But avoid



          • Asking for help, clarification, or responding to other answers.

          • Making statements based on opinion; back them up with references or personal experience.


          Use MathJax to format equations. MathJax reference.


          To learn more, see our tips on writing great answers.





          Some of your past answers have not been well-received, and you're in danger of being blocked from answering.


          Please pay close attention to the following guidance:


          • Please be sure to answer the question. Provide details and share your research!

          But avoid



          • Asking for help, clarification, or responding to other answers.

          • Making statements based on opinion; back them up with references or personal experience.


          To learn more, see our tips on writing great answers.




          draft saved


          draft discarded














          StackExchange.ready(
          function () {
          StackExchange.openid.initPostLogin('.new-post-login', 'https%3a%2f%2fmath.stackexchange.com%2fquestions%2f3012140%2fmoment-of-inertia-of-a-solid-right-circular-cone%23new-answer', 'question_page');
          }
          );

          Post as a guest















          Required, but never shown





















































          Required, but never shown














          Required, but never shown












          Required, but never shown







          Required, but never shown

































          Required, but never shown














          Required, but never shown












          Required, but never shown







          Required, but never shown







          Popular posts from this blog

          How do I know what Microsoft account the skydrive app is syncing to?

          When does type information flow backwards in C++?

          Grease: Live!